2022 AMC 10B Problems/Problem 13

Revision as of 19:26, 17 November 2022 by Trex226 (talk | contribs) (Created page with "==Solution== Let the two primes be <math>a</math> and <math>b</math>. We would have <math>a-b=2</math> and <math>a^{3}-b^{3}=31106</math>")
(diff) ← Older revision | Latest revision (diff) | Newer revision → (diff)

Solution

Let the two primes be $a$ and $b$. We would have $a-b=2$ and $a^{3}-b^{3}=31106$